Simplify
2x + 5x^3 + 3x - 4x^3 ​

Answers

Answer 1

Answer:

x³ + 5x

Step-by-step explanation:

2x+5x3+3x−4x3

=2x+5x3+3x+−4x3

Combine Like Terms:

=2x+5x3+3x+−4x3

=(5x3+−4x3)+(2x+3x)

=x3+5x

Answer 2

Answer:

[tex]\large \boxed{x^3+5x}[/tex]

Step-by-step explanation:

[tex]2x + 5x^3 + 3x - 4x^3[/tex]

[tex]\sf Group \ like \ terms.[/tex]

[tex]5x^3 - 4x^3+2x+3x[/tex]

[tex]\sf Combine \ like \ terms.[/tex]

[tex]x^3+5x[/tex]


Related Questions

Pls answer I really need help
Brainlist and thank you will be the reward thank you so much!!!

Answers

Answer:

0.667 ✅

Step-by-step explanation:

This is best solved using a proportion.

The formula is soy/vinegar = soy/vinegar where one of these is a variable.

Here we have:

[tex]\frac{150}{100} = \frac{1}{x}[/tex]

Now, we solve this by cross multiplying.

150x = 100

Dividing both sides by x, we get x = 2/3 or about 0.667.

Checking:

[tex]\frac{150}{100} = \frac{1}{0.667}[/tex]

1.5 = 1.5 ✅

I'm always happy to help :)

Line m and point P are shown below. Part A: Using a compass and straightedge, construct line n parallel to line m and passing through point P. Leave all construction marks. Part B: Explain the process that you used to construct line n.

Answers

Answer:

  see the attached

Step-by-step explanation:

In the attachment, we will refer to the circles, bottom-to-top, as circles 1, 2 and 3. The black points of intersection, bottom-to-top, will be referred to by the letters A, B, C, D. The transversal line through the white point (W) and pink point (P) will be line q.

Step 1. Draw line q through point P so it intersects line m at some convenient point. Label that point W.

Step 2. Choose an arbitrary radius for your compass. Here, we have chosen it to be the length WB. It happens to be less than half the length of WP, but that is not a requirement.

Step 3. Draw an arc of the chosen radius centered at W and intersecting line q and line m. Label the intersection points A (on line m) and B (on line q). These intersection points are on circle 1.

Step 4. Draw an arc of the same radius centered at P. It should be a long enough arc that it would intersect the proposed line parallel to m. Label the intersection point on line q with label C. This intersection point is on circle 3.

Step 5. Adjust the compass width (radius) to the distance from A to B. This is the radius of circle 2.

Step 6. Draw an arc centered at C so that it intersects the arc of Step 4. This is circle 2, and you want it to intersect circle 3. Label that point of intersection D.

Step 7. Draw line PD parallel to m.

_____

The point of the construction is to create congruent alternate interior angles AWB and CPD, so that lines AW and PD are parallel.

Kelly bought a crate of floor tiles for $95.94. The crate had 6 boxes of floor tiles. Each box contained 20 floor tiles.

Write and solve an equation to determine the cost per box, b. Then write and solve a second equation to determine the cost per tile, t, to the nearest cent.

Answers

Answer:

$1.60 a crate

Step-by-step explanation:

t= 95.94/(6x20)

(6x20)= 60

95.94/60

$1.60

Answer:

Step-by-step explanation:

i) Cost per box = cost of a crate ÷ Number of boxes in the crate

b = 95.94 ÷ 6

b = $ 15.99

ii) Cost per tile = Cost per box ÷ Number of tiles in a box

 t = b ÷ 20

 t = 15.99 ÷20

t = $ 0.7995

can u help me with this?​

Answers

Answer:  Yes. The sales tax is 5% which equals $4.20 for $84

Step-by-step explanation:

[tex]\dfrac{0.60}{12}=0.05\qquad \rightarrow 5\%\\\\\\\dfrac{1.20}{24}=0.05\qquad \rightarrow 5\%\\\\\\\dfrac{1.80}{36}=0.05\qquad \rightarrow 5\%\\\\\\\dfrac{2.40}{48}=0.05\qquad \rightarrow 5\%[/tex]

The sales tax rate is proportional for the values in the table.

$84 x 0.05 = $4.20

The sales tax on a purchase of $84 is $4.20

3 sides of the triangle are consecutive odd numbers. What is the smallest possible perimeter of the triangle ?

Answers

The sides are 3, 5, and 7 because 7 is less than 3 + 5, so it equals a triangle.

The smallest possible perimeter of such triangle would be 8.

What is triangle inequality theorem?

Triangle inequality theorem of a triangle says that the sum of any of the two sides of a triangle is always greater than the third side.

Suppose a, b and c are the three sides of a triangle. Thus according to this theorem,

[tex](a+b) > c\\(b+c) > a\\(c+a) > b[/tex]

Given; 3 sides of the triangle are consecutive odd numbers.

The sides are 3, 5, and 7 because 7 is less than 3 + 5 so it equals a triangle.

The smallest consecutive odd numbers are 1, 3 and 5

Therefore, the smallest possible perimeter of such triangle = 8

Learn more about triangle inequality theorem here:

https://brainly.com/question/342881

#SPJ2

What does this mean? 20 points + brainleist! What other angle measures or side lengths can you determine using these added figures? List all the concepts and facts you use.

Answers

Step-by-step explanation:

The question has asked you to have another concepts or facts that can be used to find the other angles and measure of side length.

by looking the fig we can determine various sides and angles as all are joined together.

Brian invested his savings in two investment funds. The $8000 that he invested in Fund A returned a 4% profit. The amount that he invested in Fund B returned a 1% profit. How much did he invest in Fund B, if both funds together returned a 2% profit?

Answers

Answer: Brian invested $16000 in Fund B .

Step-by-step explanation:

Let x be the amount Brian invested in Fund B.

Given, The $8000 that he invested in Fund A returned a 4% profit. The amount that he invested in Fund B returned a 1% profit.

i.e. profit on Fund A = 4% of 8000 = 0.04 ×8000 = $320

Profit on Fund B = 1% of x = 0.01x

Together they earn 1% profit, i.e. Combined profit = 2% of (8000+x)

= 0.02(8000+x)

As per question,

Combined profit=Profit on Fund A+Profit on Fund B

[tex]\Rightarrow\ 0.02(8000+x) =320+0.01x\\\\\Rightarrow\ 0.02(8000) +0.02x=320+0.01x\\\\\Rightarrow\ 160+0.02x=320+0.01x\\\\\Rightarrow\ 0.02x-0.01x=320-160\\\\\Rightarrow\ 0.01x=160\\\\\Rightarrow\ x=\dfrac{160}{0.01}\\\\\Rightarrow\ x=16000[/tex]

Hence, Brian invested $16000 in Fund B .

If a polygon has an area of 10 cm² and is dilated by a factor of 2, what will be the area of the dilated polygon?

Answers

Area depends on the product of sides,

so if the sides are shortened by a factor of 2, area will reduce by a factor of 4. (2×2)

new area = 10/4=2.5 cm²

[fill in the blank]
In this figure,AB and CD are parallel.

AB is perpendicular to line segment_____. If the length of EF is a units, then the length of GH is_____units.

Answers

Answer:

1. GH

2. a

Step-by-step explanation:

Perpendicular: When 2 lines meet at 90 degrees

1. It is line segment GH because AB and GH meet at a 90 degree angle (since there is a box at angle GHF indicating that it is 90 degrees)

2. It has to be a units because it is a rectangle where the top and bottom are congruent and the sides are too

This is a rectangle since AB and CD are parallel and GH can be a transversal line, according to same side interior angles theorem EGH is a also 90 degrees. That means FEG is 90 degrees too because then the quadrilateral will add up to 360 degrees

There are approximately fifty five million metric tons of apples grown each year. It has been estimated that half of all the apples grown are eaten fresh. Approximately how many metric tons of apples are eaten fresh

Answers

Answer:

Twenty-seven and a half million metric tons of apples are eaten fresh.

Step-by-step explanation:

The amount of metric tons of apples that are eaten fresh is equal to the half of the total amount of metric tons. That is:

[tex]m = \frac{55,000,000\,m-ton}{2}[/tex]

[tex]m = 27,500,000\,m-ton[/tex]

Twenty-seven and a half million metric tons of apples are eaten fresh.

Review what you know about products and sums represented by rectangular area models. [5 points] Use algebra tiles to multiply (x-1)(3x+2).

Answers

(x - 1)(3x + 2)
= 3x^2 + 2x - 3x - 2
= 3x^2 - x - 2

3x^2 - x - 2

What are some ways to solve an equation?

Different ways to solve equations. We have 4 ways of solving one-step equations: Adding, Substracting, multiplication, and division. If we add the same number to both sides of an equation, both sides will remain equal.

How do you evaluate an equation?

∫ y2+y−2dy ∫ y 2 + y − 2 d y∫ 2 1 y2 +y−2dy ∫ 1 2 y 2 + y − 2 d y∫ 2 −1 y2 +y−2dy ∫ − 1 2 y 2 + y − 2 d y

= (x - 1)(3x + 2)

= 3x^2 + 2x - 3x - 2

= 3x^2 - x - 2

Learn more about equation here https://brainly.com/question/10413253

#SPJ2

HELP i don’t know how to do this

Answers

Answer:

4a

Step-by-step explanation:

4a

the top and right are a-b, but you have to add the b’s back in, so really all sides are a

a+a+a+a=4a

Answer: 4a

Step-by-step explanation: perimeter is the length of all sides added together. Every length is given combine like variables and you will get 4a+2b-2b. 2b-2b is 0 which leaves you with 4a

Find the slope of the line that passes through the points (-8,-3) and (2, 3)
0
1
3/5
5/3

Answers

Answer:

The answer is

[tex] \frac{3}{5} [/tex]

Step-by-step explanation:

To find the slope passing through two points we use the formula

[tex]m = \frac{y2 - y1}{x2 - x1} [/tex]

Where

m is the slope

( x1 , y1) and ( x2 , y2) are the points

From the question the points are

(-8,-3) and (2, 3)

So the slope is

[tex]m = \frac{3 + 3}{2 + 8} = \frac{6}{10} = \frac{3}{5} [/tex]

Hope this helps you

What is the result when the number 90 is decreased by 10%

Answers

Answer:

81

Step-by-step explanation:

First find the amount decrease

90 * 10 %

90 * .10

9

90 decreased by 9

90 -9

81

Answer:

81

Step-by-step explanation:

Turn into decimal.

10% = 0.1

Multiply

90 * 0.1 = 9

Subtract

90 - 9 = 81

Best of Luck!

math now..!! Help..?

Answers

Answer:

2p + 12

2 (p = +6) + 12 = 20

Answer:

I think its 6

Step-by-step explanation:

because you have to add 9 and 3 together then you get 12 and you have to divide 2p with 12 and you'll get 6

Combine the radicals. 2√24+5√54 A) 53√6 B) 5√6 C) 19√6 D) 93√6

Answers

Answer:

The answer is option C

Step-by-step explanation:

2√24 + 5√54

To combine the radicals first make sure the radicals have the same square root

That's

For 2√24

[tex]2 \sqrt{24} = 2 \sqrt{4 \times 6} = 2 \times 2 \sqrt{6} [/tex][tex] = 4 \sqrt{6} [/tex]For 9√54

[tex]5 \sqrt{54} = 5 \sqrt{9 \times 6} = 5 \times \sqrt{9} \times \sqrt{6} [/tex][tex] = 5 \times 3 \times \sqrt{6} [/tex][tex] = 15 \sqrt{6} [/tex]

Since they have the same square root we can combine them

That's

[tex]4 \sqrt{6} + 15 \sqrt{6} = (4 + 15) \sqrt{6} [/tex]

We have the final answer as

[tex]19 \sqrt{6} [/tex]

Hope this helps you

Write an equation for the line in the graph that passes through the points (0,4) and (12,16).

Answers

Answer:

We have,

y-y1=m(x-x1)

or,y-4=-1(x-0)

or,y-4=-x

or,x+y=4 is the required equation

Step-by-step explanation:

it it helps u ...plz mark it as brainliest

What is the solution to 7 × p = -56? A. -49 B. -8 C. 8 D. 49

Answers

Answer:

-8

Step-by-step explanation:

Hello!

What we do to one side of the equation we do to the other side

7 * p = -56

Divide both sides by 7

p = -8

The answer is -8

Hope this helps!

WILL MARK BRAINLIEST!!!!20 POINTS!!!!URGENT!!!

Answers

Answer:

the firsts third and last one

Step-by-step explanation:

hope this helped

Which equation represents the line that is perpendicular to y=3/4x+1 and passes through (-5,11)
Will give brainliest!!

Answers

Answer:

y = - [tex]\frac{4}{3}[/tex] x + [tex]\frac{13}{3}[/tex]

Step-by-step explanation:

The equation of a line in slope- intercept form is

y = mx + c ( m is the slope and c the y- intercept )

y = [tex]\frac{3}{4}[/tex] x + 1 ← is in slope- intercept form

with slope m = [tex]\frac{3}{4}[/tex]

Given a line with slope m then the slope of a line perpendicular to it is

[tex]m_{perpendicular}[/tex] = - [tex]\frac{1}{m}[/tex] = - [tex]\frac{1}{\frac{3}{4} }[/tex] = - [tex]\frac{4}{3}[/tex] , thus

y = - [tex]\frac{4}{3}[/tex] x + c ← is the partial equation

To find c substitute (- 5, 11) into the partial equation

11 = [tex]\frac{20}{3}[/tex] + c ⇒ c = 11 - [tex]\frac{20}{3}[/tex] = [tex]\frac{13}{3}[/tex]

y = - [tex]\frac{4}{3}[/tex] x + [tex]\frac{13}{3}[/tex] ← equation of perpendicular line

The equation of the line that passes through (-5, 11) and perpendicular to y = (3/4)x + 1 is

y = -2x + 1

What is an equation of a line?

The equation of a line is given by:

y = mx + c

where m is the slope of the line and c is the y-intercept.

Example:

The slope of the line y = 2x + 3 is 2.

The slope of a line that passes through (1, 2) and (2, 3) is 1.

We have,

y = (2/4)x + 1 is in the form of y = m(2)x + c

So,

m(2) = 2/4 = 1/2

The equation of the line y = m(1)x + c is perpendicular to y = (2/4)x + 1.

So,

m(1) x m(2) = -1

m(1) = -1/(1/2)

m(1) = -2

Now,

y = -2x + c passes through (-5, 11).

This means,

11 = -2 x (-5) + c

11 = 10 + c

11- 10 = c

c = 1

Thus,

The equation of the line is y = -2x + 1.

Learn more about equation of a line here:

https://brainly.com/question/23087740

#SPJ2

Suppose y varies jointly as x & z. If y = -180 when z = 15and x = -3,then find y when x = 7 and z = -5. 1 point

Answers

Answer:

y = - 140

Step-by-step explanation:

The statement

y varies jointly as x & z is written as

y = kxz

to find y when x = 7 and z = -5 we must first find the relationship between the variables

when y = - 180

z = 15

x = - 3

- 180 = k(15)(-3)

-180 = - 45k

Divide both sides by - 45

k = 4

The formula for the variation is

y = 4xz

when

x = 7

z = -5

y = 4(7)(-5)

y = 4(-35)

y = - 140

Hope this helps you

Dawn and Jackson have baseball cards in the ratio of 2:3. Together, they have a total of 60 baseball cards. How many baseball cards does each child have?

Answers

Answer:

24 and 36

Step-by-step explanation:

2x + 3x = 60

5x = 60

x = 12

Dawn has 2(12) = 24

Jackson has 3(12) = 36

Step-by-step explanation:

To find the number of baseball cards each person received we must first find the total parts

That's

2 + 3 = 5

For Dawn

Dawn's part is 2

We have

2/5 × 60

= 24 baseball cards

For Jackson

Jackson's part is 3

That's

3/5 × 60

= 36 baseball cards

Hope this helps you

Puzzle corner
Look Before You Leap!
See how long it takes you to work out the
following:
(1 x2)×(3 x 4)×(586)×(7 x 8) x (
9×0)

Answers

Answer:

0

Step-by-step explanation:

Notice that the last factor is null (9×0)

So the result will be null since any number that is multiplied by 0 equals 0.

What is the width of the rectangle shown below?
4x + 3
A = 8x2 – 10x – 12

Answers

Answer:

2x-4

Step-by-step explanation:

Area of a rectangle = Length * Width

Given parameters

Area A = 8x2 – 10x – 12

Length of the rectangle = 4x+3

Required

Width of the rectangle.

Substituting the given parameters into the formula

8x2 – 10x – 12  = (4x+3)*width

width = 8x2 – 10x – 12 /4x+3

S

Factorizing the numerator

8x² – 10x – 12

= 2(4x²-5x-6)

= 2(4x²-8x+3x-6)

= 2(4x(x-2)+3(x-2))

= 2(4x+3)(x-2)

Width = 2(4x+3)(x-2)/4x+3

Width = 2(x-2)

Width = 2x-4

Hence the width of the rectangle is 2x-4

what do you think you’d like most about working as a forensic scientist? why

Answers

Answer:

i think its very interesting and pretty cool,  because there is so much to learn  and so much to explore

i wouldn't like the fact that you have to study so much though

Step-by-step explanation:

Write in standard 7.125×10^-9

Answers

Answer:

7.125*10⁻⁹ = 0.000000007125

Step-by-step explanation:

7.125*10⁻⁹ = 0.000000007125

Can someone pls help me &THANK YOU



SLATTTT!!!

Answers

BABYYYY SOSAAAAA CASH CARTIIII

The physician orders 1500 flu injections. There were 300 shots given in September, 450 shots given in October, and 665 shots given in November. What percentage of flu vaccine has been given?

Answers

Answer:

94.33

Step-by-step explanation:

add up all numbers then input into a percentage calculator

Percentage of flu vaccine has been given 94.33%

What is percentage?

Percentage is defined as ratio expressed as a fraction of 100.

The physician have total 1500 flu injections

300 shots given in September,

450 shots given in October,

and 665 shots given in November,

Total flu vaccine has been given = 300+450+665 = 1415

Percentage of flu vaccine has been given = (1415/1500)×100%

Percentage of flu vaccine has been given = 0.9433×100%

Percentage of flu vaccine has been given = 94.33%

Hence, percentage of flu vaccine has been given 94.33%

Learn more about percentage

brainly.com/question/24159063

#SPJ2

convert the equation f(x)=1/2x^2+3x-2 to vertex form

Answers

Answer:

Step-by-step explanation:

Hello, please consider the following.

The "vertex form" is as below.

[tex]y=a(x-h)^2+k\\\\\text{Where (h, k) is the vertex of the parabola.}\\[/tex]

Let's do it!

[tex]f(x)=\dfrac{1}{2}x^2+3x-2\\\\f(x)=\dfrac{1}{2}\left(x^2+3*2*x\right) -2\\\\f(x)=\dfrac{1}{2}\left( (x+3)^2-3^2\right)-2\\\\f(x)=\dfrac{1}{2}(x+3)^2-\dfrac{9}{2}-\dfrac{4}{2}\\\\f(x)=\dfrac{1}{2}(x+3)^2-\dfrac{9+4}{2}\\\\\large \boxed{\sf \bf \ \ f(x)=\dfrac{1}{2}(x+3)^2-\dfrac{13}{2} \ \ }[/tex]

Thank you.

In the first quadrant you start at 5, 6 and move 4 units down. What point will you end up at? Thanks for your help! - Someone who's better at English than math

Answers

Answer:

(5, 2)

Step-by-step explanation:

(5, 6) go down 4 units means subtract 4 from the y

(5, 2)

The point to end up will be (5, 2).

What is Coordinates?

A pair of numbers which describe the exact position of a point on a cartesian plane by using the horizontal and vertical lines is called the coordinates.

Given that;

In the first quadrant you start at (5, 6 ) and move 4 units down.

Now,

Since, In the first quadrant you start at 5, 6 and move 4 units down.

Hence, The end up point = (5, 6 - 4)

                                       = (5, 2)

Thus, The point to end up will be (5, 2).

Learn more about the coordinate visit:

https://brainly.com/question/18269861

#SPJ5

Other Questions
herefore you have no excuse, everyone of you who passes judgment, for in that which you judge another, you condemn yourself; for you who judge practice the same things. This passage means that people are equal because they are all Which word best completes this sentence? Me gusta ____________ porque me ahorro dinero. A. el auto B. el taxi C. el tren D. el metro E. la bicicleta what is mean by stakeholder The Clifford Corporation has announced a rights offer to raise $17 million for a new journal, the Journal of Financial Excess. This journal will review potential articles after the author pays a nonrefundable reviewing fee of $6,000 per page. The stock currently sells for $42 per share, and there are 2.9 million shares outstanding. a. What is the maximum possible subscription price? What is the minimum? (Leave no cells blank - be certain to enter "0" wherever required.) b. If the subscription price is set at $34 per share, how many shares must be sold? How many rights will it take to buy one share? (Do not round intermediate calculations. Round your rights needed answer to 2 decimal places, e.g., 32.16.) c. What is the ex-rights price? What is the value of a right? (Do not round intermediate calculations and round your answers to 2 decimal places, e.g., 32.16.) d. A shareholder with 2,000 shares before the offering has no desire (or money) to buy additional shares offered as rights. What is his portfolio value before and after the rights offer? (Do not round intermediate calculations and round your answers to nearest whole number, e.g., 32.) Literary works Communication Composition Literature Forensics Light of wavelength 500 nm falls on two slits spaced 0.2 mm apart. If the spacing between the first and third dark fringes is to be 4.0 mm, what is the distance from the slits to a screen? What is the main idea of William Pitts speech about the stamp act Children that are born with a disease that impairs the function of B cells and cannot make antibodies have difficulty mounting an immune response to which type(s) of infection? Celine is Drakes granddaughter. Her age is 4 years greater than of Drakes age. If Celine is 28 years old, how old is Drake? Dtermine the answer to (5) + 4 and explain the steps using a number line What is the range of g? Factor completely 2c5 + 44c4 + 242c3. 2c3(c + 11)2 2(c + 11)2 2c3(c + 11)(c 11) 2c3(c2 + 22c + 121) If you were given a personality test as part of an employment application process, would you answer the questions honestly or would you attempt to answer the questions based upon your image of "correct" way to answer? what implications does your response has for the validity of personality testing? 5. Suppose that a firm is in an industry which has a very rapid rate of growth (in sales and output), and is characterized by technological change and innovation. Firms attempt to maximize profits causing new firms to enter the industry attracted by profit potential. The result is that profits are competed away, leading to even greater innovation and change. Is there a limit to this continuous change Find X using the Angle Sum Theorem 2) In 1000 sq. meter of land a farmer cultivated 765 kg of rice with the wastage of 23.5%. I) Find the weight of the wastage. II) Find the weight and percentage of rice cultivated. 3) If the area has been increased 40 times in size, how much rice will be cultivated (excluding the wastag Find the domain and the range of the relation.Find the domain of the relation. Select the correct choice below and fill in the answer box tocomplete your choice.O A. The domain is _(Type your answer in interval notation.)B. The domain is {_}(Type an integer or a fraction. Use a comma to separate answers as needed.)Find the range of the relation. Select the correct choice below and fill in the answer box tocomplete your choice.O A. The range is _(Type an integer or a fraction. Use a comma to separate answers as needed.)OB. The range is {_} Solve for x.13(x-3) = 39x=1x=4x=6x= 10 Mr. Bauer is 49 years old, but eighteen months ago he was declared disabled by the Social Security Administration and has been receiving disability payments. He is wondering whether he can obtain coverage under Medicare. What should you tell him? write a letter to your assembly member asking him to help the maintenance of your school structure.